(10 intermediate revisions by 2 users not shown)
Line 1: Line 1:
 +
[[Category:ECE]]
 +
[[Category:QE]]
 +
[[Category:CNSIP]]
 +
[[Category:problem solving]]
 +
[[Category:automatic control]]
 +
[[Category:optimization]]
 +
 
= [[ECE PhD Qualifying Exams|ECE Ph.D. Qualifying Exam]] in "Automatic Control" (AC)  =
 
= [[ECE PhD Qualifying Exams|ECE Ph.D. Qualifying Exam]] in "Automatic Control" (AC)  =
  
= Question 3, Part 3, August 2011  =
+
= [[ECE-QE_AC3-2011|Question 3, August 2011]],  Part 3 =
  
 
:[[ECE-QE AC3-2011 solusion-1|Part 1]],[[ECE-QE AC3-2011 solusion-2|2]],[[ECE-QE_AC3-2011_solusion-3|3]],[[ECE-QE AC3-2011 solusion-4|4]],[[ECE-QE AC3-2011 solusion-5|5]]
 
:[[ECE-QE AC3-2011 solusion-1|Part 1]],[[ECE-QE AC3-2011 solusion-2|2]],[[ECE-QE_AC3-2011_solusion-3|3]],[[ECE-QE AC3-2011 solusion-4|4]],[[ECE-QE AC3-2011 solusion-5|5]]
Line 19: Line 26:
 
----
 
----
  
'''Theory'''
+
'''Theorem:'''  
  
The '''Fundamental Theorem of Linear Programming''' that one of the basic feasible solutions is an optimal solution. 
+
The '''Fundamental Theorem of Linear Programming''' that one of the basic feasible solutions is an optimal solution.   
  
 
----
 
----
Line 34: Line 41:
 
<math>\Rightarrow x_{2}-x_{3}=4</math>  
 
<math>\Rightarrow x_{2}-x_{3}=4</math>  
  
<math>\text{It is equivalent to  min }  x_{1}+3x_{2}-4x_{3}
+
<span class="texhtml">It is equivalent to  min ''x''<sub>1</sub> + 3''x''<sub>2</sub> − 4''x''<sub>3</sub> = 5 2''x''<sub>2</sub> + ''x''<sub>3</sub> + 3''x''<sub>2</sub> − 4''x''<sub>3</sub> = ''x''<sub>2</sub> − 3''x''<sub>3</sub> + 5,</span>
=5-2x_{2}+x_{3}+3x_{2}-4x_{3} = x_{2}-3x_{3}+5, x_{2}\geq0, x_{3}\leq0</math>  
+
 
 +
&nbsp; &nbsp; &nbsp; &nbsp; &nbsp; &nbsp; &nbsp; &nbsp; &nbsp; &nbsp; &nbsp; &nbsp; &nbsp; &nbsp; &nbsp; &nbsp;&nbsp;<math>x_{2}\geq0, x_{3}\leq0</math>  
  
 
<math>x_{2}-3x_{3}+5 = x_{2}-x_{3}-2x_{3}+5=9-2x_{3}\geq9</math>&nbsp; &nbsp;<math>\color{green}  \text{constrain:  } x_{3}\leq0 \Rightarrow x_{3}=0</math>  
 
<math>x_{2}-3x_{3}+5 = x_{2}-x_{3}-2x_{3}+5=9-2x_{3}\geq9</math>&nbsp; &nbsp;<math>\color{green}  \text{constrain:  } x_{3}\leq0 \Rightarrow x_{3}=0</math>  
Line 47: Line 55:
 
x_{2}=4\\  
 
x_{2}=4\\  
 
x_{3}=0
 
x_{3}=0
\end{matrix}\right.</math>  
+
\end{matrix}\right.</math>&nbsp; &nbsp; &nbsp; &nbsp; &nbsp;&nbsp;<math>\color{green} \text{The answer is correct.}</math>  
  
<math>\color{green} \text{This solution is not using the Fundamental Theorem of Linear Programming}</math>  
+
<math>\color{green} \text{But, this solution is NOT using the Fundamental Theorem of LP.}</math>  
  
 
----
 
----
Line 84: Line 92:
 
&nbsp; &nbsp; &nbsp; &nbsp;&nbsp;<math>x^{\left( 1 \right)}= \begin{bmatrix}
 
&nbsp; &nbsp; &nbsp; &nbsp;&nbsp;<math>x^{\left( 1 \right)}= \begin{bmatrix}
 
-3 & 4 & 0
 
-3 & 4 & 0
\end{bmatrix}^{T} \text{ is BFS. } f_{1}=-9</math>  
+
\end{bmatrix}^{T} \text{ is a BFS. } f_{1}=-9</math>  
  
 
<math>\text{The second basis candidate is } \begin{pmatrix}
 
<math>\text{The second basis candidate is } \begin{pmatrix}
Line 101: Line 109:
 
&nbsp; &nbsp; &nbsp; &nbsp;&nbsp;<math>x^{\left( 2 \right)}= \begin{bmatrix}
 
&nbsp; &nbsp; &nbsp; &nbsp;&nbsp;<math>x^{\left( 2 \right)}= \begin{bmatrix}
 
0 & 1 & -3
 
0 & 1 & -3
\end{bmatrix}^{T} \text{ is BFS. } f_{2}=-15</math>  
+
\end{bmatrix}^{T} \text{ is a BFS. } f_{2}=-15</math>  
  
 
<math>\text{The third basis candidate is } \begin{pmatrix}
 
<math>\text{The third basis candidate is } \begin{pmatrix}
Line 118: Line 126:
 
&nbsp; &nbsp; &nbsp; &nbsp;&nbsp;<math>x^{\left( 2 \right)}= \begin{bmatrix}
 
&nbsp; &nbsp; &nbsp; &nbsp;&nbsp;<math>x^{\left( 2 \right)}= \begin{bmatrix}
 
1 & 0 & -5
 
1 & 0 & -5
\end{bmatrix}^{T} \text{ is BFS. } f_{3}=-21</math>  
+
\end{bmatrix}^{T} \text{ is a BFS. } f_{3}=-21</math>  
  
 
<math>\because f_{1}>f_{2}>f_{3} \text{ , where } f_{1}=-9 \text{ is maximal.}</math>  
 
<math>\because f_{1}>f_{2}>f_{3} \text{ , where } f_{1}=-9 \text{ is maximal.}</math>  
Line 126: Line 134:
 
\end{bmatrix}  \text{ with objective value } -9</math>  
 
\end{bmatrix}  \text{ with objective value } -9</math>  
  
<math>\color{green} \text{This solution use the Fundamental Theorem of Linear Programming. We generate all possible basic feasible solutions and select from them the optimal one.}</math>
+
<math>\color{green} \text{This solution use the Fundamental Theorem of Linear Programming. }</math>
 +
 
 +
<math>\color{green} \text{All possible basic feasible solutions are generated }</math>
 +
 
 +
&nbsp; &nbsp; &nbsp; &nbsp; &nbsp; &nbsp; &nbsp;&nbsp;<math>\color{green} \text{ and from which the optimal one is selected.}</math>  
  
 
----
 
----
  
<math>\color{blue} \text{Related Problem: Solve following linear programming problem,}</math>  
+
<math>\color{blue} \text{Related Problem: Solve the following linear programming problem,}</math>  
  
 
minimize &nbsp;&nbsp;<span class="texhtml">3''x''<sub>1</sub> + ''x''<sub>2</sub> + ''x''<sub>3</sub></span>  
 
minimize &nbsp;&nbsp;<span class="texhtml">3''x''<sub>1</sub> + ''x''<sub>2</sub> + ''x''<sub>3</sub></span>  
Line 140: Line 152:
 
&nbsp; &nbsp; &nbsp; &nbsp; &nbsp; &nbsp; &nbsp; &nbsp; &nbsp;&nbsp;<math>x_{1}\geq0,x_{2}\geq0,x_{3}\geq0,</math>  
 
&nbsp; &nbsp; &nbsp; &nbsp; &nbsp; &nbsp; &nbsp; &nbsp; &nbsp;&nbsp;<math>x_{1}\geq0,x_{2}\geq0,x_{3}\geq0,</math>  
  
<math>\color{blue}\text{Solution}</math>  
+
<math>\color{blue}\text{Solution :}</math>  
  
 
<span class="texhtml">The equality constraints can be represented in the form ''A'''x'''''<b> = ''b'',</b></span>'''<br> '''  
 
<span class="texhtml">The equality constraints can be represented in the form ''A'''x'''''<b> = ''b'',</b></span>'''<br> '''  
Line 163: Line 175:
 
&nbsp; &nbsp; &nbsp; &nbsp; &nbsp;<math>\text{The corresponding basic solution is } x^{\left( 1 \right)}= \begin{bmatrix}
 
&nbsp; &nbsp; &nbsp; &nbsp; &nbsp;<math>\text{The corresponding basic solution is } x^{\left( 1 \right)}= \begin{bmatrix}
 
4 & 2 & 0
 
4 & 2 & 0
\end{bmatrix}^{T} \text{ is BFS.}</math><br>  
+
\end{bmatrix}^{T} \text{ is a BFS.}</math><br>  
  
 
&nbsp; &nbsp; &nbsp; &nbsp; &nbsp;<span class="texhtml">The objective function value is ''f''<sub>1</sub> = 14</span>  
 
&nbsp; &nbsp; &nbsp; &nbsp; &nbsp;<span class="texhtml">The objective function value is ''f''<sub>1</sub> = 14</span>  
Line 172: Line 184:
 
\end{pmatrix}</math><br>  
 
\end{pmatrix}</math><br>  
  
&nbsp; &nbsp; &nbsp; &nbsp; &nbsp;<math>\text{The corresponding basic solution is } x^{\left( 2 \right)}= \begin{bmatrix}
+
&nbsp; &nbsp; &nbsp; &nbsp;&nbsp;<math>\text{The corresponding basic solution is } x^{\left( 2 \right)}= \begin{bmatrix}
 
6 & 0 & -2
 
6 & 0 & -2
\end{bmatrix}^{T} \text{, which is NOT a BFS.}</math>&nbsp; &nbsp; &nbsp; &nbsp;
+
\end{bmatrix}^{T} \text{, which is NOT a BFS.}</math>&nbsp; &nbsp; &nbsp; &nbsp;  
  
 
<math>\text{The third basis candidate is } \begin{pmatrix}
 
<math>\text{The third basis candidate is } \begin{pmatrix}
Line 205: Line 217:
 
----
 
----
  
<br> [[ECE PhD Qualifying Exams|Back to ECE Qualifying Exams (QE) page]]
+
<br> [[ECE PhD Qualifying Exams|Back to ECE Qualifying Exams (QE) page]]
 
+
[[Category:ECE]] [[Category:QE]] [[Category:Automatic_Control]] [[Category:Problem_solving]]
+

Latest revision as of 10:10, 13 September 2013


ECE Ph.D. Qualifying Exam in "Automatic Control" (AC)

Question 3, August 2011, Part 3

Part 1,2,3,4,5

 $ \color{blue}\text{3. } \left( \text{20 pts} \right) \text{ Solve the following linear program, } $

maximize    − x1 − 3x2 + 4x3

subject to    x1 + 2x2x3 = 5

                   2x1 + 3x2x3 = 6

                   $ x_{1} \text{ free, } x_{2}\geq0, x_{3}\leq0. $


Theorem:

The Fundamental Theorem of Linear Programming that one of the basic feasible solutions is an optimal solution. 


$ \color{blue}\text{Solution 1:} $

$ \left.\begin{matrix} x_{1}+2x_{2}-x_{3}=5 \\ 2x_{1}+3x_{2}-x_{3}=6 \end{matrix}\right\}\Rightarrow x_{1}=5-2x_{2}+x_{3}=3-\frac{3}{2}x_{2}+\frac{1}{2}x_{3} $

$ \Rightarrow x_{2}-x_{3}=4 $

It is equivalent to min x1 + 3x2 − 4x3 = 5 − 2x2 + x3 + 3x2 − 4x3 = x2 − 3x3 + 5,

                                $ x_{2}\geq0, x_{3}\leq0 $

$ x_{2}-3x_{3}+5 = x_{2}-x_{3}-2x_{3}+5=9-2x_{3}\geq9 $   $ \color{green} \text{constrain: } x_{3}\leq0 \Rightarrow x_{3}=0 $

$ \text{Equivalently, } -x_{1}-3x_{2}+4x_{3}\leq-9 $

$ \text{Equality is satisfied when } x_{3}=0, x_{2} =4+0=4, x_{1}=5-2\times4=-3 $

$ \Rightarrow \left\{\begin{matrix} x_{1}=-3\\ x_{2}=4\\ x_{3}=0 \end{matrix}\right. $          $ \color{green} \text{The answer is correct.} $

$ \color{green} \text{But, this solution is NOT using the Fundamental Theorem of LP.} $


$ \color{blue}\text{Solution 2:} $

One of the basic feasible solution is an optimal solution.
The equality constraints can be represented in the form Ax = b,

$ A=\begin{bmatrix} 1 & 2 & -1\\ 2 & 3 & -1 \end{bmatrix}=\begin{bmatrix} a_{1}& a_{2}& a_{3} \end{bmatrix}; b=\begin{bmatrix} 5\\ 6 \end{bmatrix} $

$ \text{The first basis candidate is } \begin{pmatrix} a_{1} & a_{2} \end{pmatrix} $

        $ \left [ A|b \right ] =\begin{bmatrix} 1 & 2 & -1 & 5 \\ 2 & 3 & -1 & 6 \end{bmatrix} = \cdots = \begin{bmatrix} 1 & 0 & 1 & -3 \\ 0 & 1 & -1 & 4 \end{bmatrix} $

        $ x^{\left( 1 \right)}= \begin{bmatrix} -3 & 4 & 0 \end{bmatrix}^{T} \text{ is a BFS. } f_{1}=-9 $

$ \text{The second basis candidate is } \begin{pmatrix} a_{2} & a_{3} \end{pmatrix} $

        $ \left [ A|b \right ] =\begin{bmatrix} 1 & 2 & -1 & 5 \\ 2 & 3 & -1 & 6 \end{bmatrix} = \cdots = \begin{bmatrix} 1 & 0 & 1 & -3 \\ 1 & 1 & 0 & 1 \end{bmatrix} $

        $ x^{\left( 2 \right)}= \begin{bmatrix} 0 & 1 & -3 \end{bmatrix}^{T} \text{ is a BFS. } f_{2}=-15 $

$ \text{The third basis candidate is } \begin{pmatrix} a_{1} & a_{3} \end{pmatrix} $

        $ \left [ A|b \right ] =\begin{bmatrix} 1 & 2 & -1 & 5 \\ 2 & 3 & -1 & 6 \end{bmatrix} = \cdots = \begin{bmatrix} 0 & 1 & 1 & 4 \\ 1 & 1 & 0 & 1 \end{bmatrix} $

        $ x^{\left( 2 \right)}= \begin{bmatrix} 1 & 0 & -5 \end{bmatrix}^{T} \text{ is a BFS. } f_{3}=-21 $

$ \because f_{1}>f_{2}>f_{3} \text{ , where } f_{1}=-9 \text{ is maximal.} $

$ \therefore \text{The optimal solution is } x^{*}=\begin{bmatrix} -3 & 4 & 0 \end{bmatrix} \text{ with objective value } -9 $

$ \color{green} \text{This solution use the Fundamental Theorem of Linear Programming. } $

$ \color{green} \text{All possible basic feasible solutions are generated } $

              $ \color{green} \text{ and from which the optimal one is selected.} $


$ \color{blue} \text{Related Problem: Solve the following linear programming problem,} $

minimize   3x1 + x2 + x3

subject to  x1 + x3 = 4

                  x2x3 = 2

                  $ x_{1}\geq0,x_{2}\geq0,x_{3}\geq0, $

$ \color{blue}\text{Solution :} $

The equality constraints can be represented in the form Ax = b,

$ A= \begin{bmatrix} 1 & 0 & 1\\ 0 & 1 & -1 \end{bmatrix}=\begin{bmatrix} a_{1}& a_{2}& a_{3} \end{bmatrix}; b=\begin{bmatrix} 4\\ 2 \end{bmatrix} $

$ \text{The first basis candidate is } \begin{pmatrix} a_{1} & a_{2} \end{pmatrix} $

         $ \text{The corresponding basic solution is } x^{\left( 1 \right)}= \begin{bmatrix} 4 & 2 & 0 \end{bmatrix}^{T} \text{ is a BFS.} $

         The objective function value is f1 = 14

$ \text{The second basis candidate is } \begin{pmatrix} a_{1} & a_{3} \end{pmatrix} $

        $ \text{The corresponding basic solution is } x^{\left( 2 \right)}= \begin{bmatrix} 6 & 0 & -2 \end{bmatrix}^{T} \text{, which is NOT a BFS.} $       

$ \text{The third basis candidate is } \begin{pmatrix} a_{2} & a_{3} \end{pmatrix} $

         $ \text{The corresponding basic solution is } x^{\left( 3 \right)}= \begin{bmatrix} 0 & 6 & 4 \end{bmatrix}^{T} \text{, which is a BFS.} $

         The objective function value is f3 = 10

$ \therefore \text{ the optimal solution is } x^{ * }= \begin{bmatrix} 0 & 6 & 4 \end{bmatrix}^{T} $



Automatic Control (AC)- Question 3, August 2011

Go to



Back to ECE Qualifying Exams (QE) page

Alumni Liaison

Ph.D. on Applied Mathematics in Aug 2007. Involved on applications of image super-resolution to electron microscopy

Francisco Blanco-Silva